Galois group of $x^3-x^2-4$












4














In determining the Galois group of the polynomial $p(x) = x^3-x^2-4,$ I concluded that is must be the Klein-$4$ group as follows. First, $p(x) = (x-2)(x^2+x+2)$ and the roots of the irreducible quadratic $x^2+x+2$ are:
$$x_{1,2} = dfrac{-1+sqrt{-7}}{2}.$$ Therefore, the splitting field of $p(x)$ is
$mathbb{Q}(sqrt{7}, i).$ Since this is a biquadratic extension and none of $i, sqrt{7}$ and $sqrt{7}i$ are squares, the Galois group is then Klein-$4$ group.



However, I found two different answers that disagree with mine. First is from the Dummit and Foote. Specifically, on page 612 it states that:




If the cubic polynomial is reducible and it splits to a linear factor and an irreducible quadratic, it's Galois group is group of order $2.$




The second source is here, where it proceeds to conclude that the polynomial is irreducible and also its Galois group is $S_3,$ on page $5.$



What is the correct answer here?










share|cite|improve this question


















  • 4




    Is the splitting field not $mathbb Q(sqrt{7}i)$?
    – Cheerful Parsnip
    Jan 3 at 1:59






  • 1




    Well, I agree 2 is a root. Therefore Dummit and Foote are right. The problem with your answer is that you've got the splitting field wrong. See Cheerful Parsnip's comment above.
    – jgon
    Jan 3 at 2:00








  • 1




    @CheerfulParsnip, actually you are right. I don't know what I was thinking. $p(x)$ is the minimal polynomial of its roots and so the group must be of order $2$ too.
    – dezdichado
    Jan 3 at 2:03










  • It's all good. That's how we learn.
    – Cheerful Parsnip
    Jan 3 at 2:24










  • @CheerfulParsnip maybe you should write an answer.
    – Kenny Lau
    2 days ago
















4














In determining the Galois group of the polynomial $p(x) = x^3-x^2-4,$ I concluded that is must be the Klein-$4$ group as follows. First, $p(x) = (x-2)(x^2+x+2)$ and the roots of the irreducible quadratic $x^2+x+2$ are:
$$x_{1,2} = dfrac{-1+sqrt{-7}}{2}.$$ Therefore, the splitting field of $p(x)$ is
$mathbb{Q}(sqrt{7}, i).$ Since this is a biquadratic extension and none of $i, sqrt{7}$ and $sqrt{7}i$ are squares, the Galois group is then Klein-$4$ group.



However, I found two different answers that disagree with mine. First is from the Dummit and Foote. Specifically, on page 612 it states that:




If the cubic polynomial is reducible and it splits to a linear factor and an irreducible quadratic, it's Galois group is group of order $2.$




The second source is here, where it proceeds to conclude that the polynomial is irreducible and also its Galois group is $S_3,$ on page $5.$



What is the correct answer here?










share|cite|improve this question


















  • 4




    Is the splitting field not $mathbb Q(sqrt{7}i)$?
    – Cheerful Parsnip
    Jan 3 at 1:59






  • 1




    Well, I agree 2 is a root. Therefore Dummit and Foote are right. The problem with your answer is that you've got the splitting field wrong. See Cheerful Parsnip's comment above.
    – jgon
    Jan 3 at 2:00








  • 1




    @CheerfulParsnip, actually you are right. I don't know what I was thinking. $p(x)$ is the minimal polynomial of its roots and so the group must be of order $2$ too.
    – dezdichado
    Jan 3 at 2:03










  • It's all good. That's how we learn.
    – Cheerful Parsnip
    Jan 3 at 2:24










  • @CheerfulParsnip maybe you should write an answer.
    – Kenny Lau
    2 days ago














4












4








4







In determining the Galois group of the polynomial $p(x) = x^3-x^2-4,$ I concluded that is must be the Klein-$4$ group as follows. First, $p(x) = (x-2)(x^2+x+2)$ and the roots of the irreducible quadratic $x^2+x+2$ are:
$$x_{1,2} = dfrac{-1+sqrt{-7}}{2}.$$ Therefore, the splitting field of $p(x)$ is
$mathbb{Q}(sqrt{7}, i).$ Since this is a biquadratic extension and none of $i, sqrt{7}$ and $sqrt{7}i$ are squares, the Galois group is then Klein-$4$ group.



However, I found two different answers that disagree with mine. First is from the Dummit and Foote. Specifically, on page 612 it states that:




If the cubic polynomial is reducible and it splits to a linear factor and an irreducible quadratic, it's Galois group is group of order $2.$




The second source is here, where it proceeds to conclude that the polynomial is irreducible and also its Galois group is $S_3,$ on page $5.$



What is the correct answer here?










share|cite|improve this question













In determining the Galois group of the polynomial $p(x) = x^3-x^2-4,$ I concluded that is must be the Klein-$4$ group as follows. First, $p(x) = (x-2)(x^2+x+2)$ and the roots of the irreducible quadratic $x^2+x+2$ are:
$$x_{1,2} = dfrac{-1+sqrt{-7}}{2}.$$ Therefore, the splitting field of $p(x)$ is
$mathbb{Q}(sqrt{7}, i).$ Since this is a biquadratic extension and none of $i, sqrt{7}$ and $sqrt{7}i$ are squares, the Galois group is then Klein-$4$ group.



However, I found two different answers that disagree with mine. First is from the Dummit and Foote. Specifically, on page 612 it states that:




If the cubic polynomial is reducible and it splits to a linear factor and an irreducible quadratic, it's Galois group is group of order $2.$




The second source is here, where it proceeds to conclude that the polynomial is irreducible and also its Galois group is $S_3,$ on page $5.$



What is the correct answer here?







abstract-algebra field-theory galois-theory






share|cite|improve this question













share|cite|improve this question











share|cite|improve this question




share|cite|improve this question










asked Jan 3 at 1:42









dezdichado

6,2551929




6,2551929








  • 4




    Is the splitting field not $mathbb Q(sqrt{7}i)$?
    – Cheerful Parsnip
    Jan 3 at 1:59






  • 1




    Well, I agree 2 is a root. Therefore Dummit and Foote are right. The problem with your answer is that you've got the splitting field wrong. See Cheerful Parsnip's comment above.
    – jgon
    Jan 3 at 2:00








  • 1




    @CheerfulParsnip, actually you are right. I don't know what I was thinking. $p(x)$ is the minimal polynomial of its roots and so the group must be of order $2$ too.
    – dezdichado
    Jan 3 at 2:03










  • It's all good. That's how we learn.
    – Cheerful Parsnip
    Jan 3 at 2:24










  • @CheerfulParsnip maybe you should write an answer.
    – Kenny Lau
    2 days ago














  • 4




    Is the splitting field not $mathbb Q(sqrt{7}i)$?
    – Cheerful Parsnip
    Jan 3 at 1:59






  • 1




    Well, I agree 2 is a root. Therefore Dummit and Foote are right. The problem with your answer is that you've got the splitting field wrong. See Cheerful Parsnip's comment above.
    – jgon
    Jan 3 at 2:00








  • 1




    @CheerfulParsnip, actually you are right. I don't know what I was thinking. $p(x)$ is the minimal polynomial of its roots and so the group must be of order $2$ too.
    – dezdichado
    Jan 3 at 2:03










  • It's all good. That's how we learn.
    – Cheerful Parsnip
    Jan 3 at 2:24










  • @CheerfulParsnip maybe you should write an answer.
    – Kenny Lau
    2 days ago








4




4




Is the splitting field not $mathbb Q(sqrt{7}i)$?
– Cheerful Parsnip
Jan 3 at 1:59




Is the splitting field not $mathbb Q(sqrt{7}i)$?
– Cheerful Parsnip
Jan 3 at 1:59




1




1




Well, I agree 2 is a root. Therefore Dummit and Foote are right. The problem with your answer is that you've got the splitting field wrong. See Cheerful Parsnip's comment above.
– jgon
Jan 3 at 2:00






Well, I agree 2 is a root. Therefore Dummit and Foote are right. The problem with your answer is that you've got the splitting field wrong. See Cheerful Parsnip's comment above.
– jgon
Jan 3 at 2:00






1




1




@CheerfulParsnip, actually you are right. I don't know what I was thinking. $p(x)$ is the minimal polynomial of its roots and so the group must be of order $2$ too.
– dezdichado
Jan 3 at 2:03




@CheerfulParsnip, actually you are right. I don't know what I was thinking. $p(x)$ is the minimal polynomial of its roots and so the group must be of order $2$ too.
– dezdichado
Jan 3 at 2:03












It's all good. That's how we learn.
– Cheerful Parsnip
Jan 3 at 2:24




It's all good. That's how we learn.
– Cheerful Parsnip
Jan 3 at 2:24












@CheerfulParsnip maybe you should write an answer.
– Kenny Lau
2 days ago




@CheerfulParsnip maybe you should write an answer.
– Kenny Lau
2 days ago










1 Answer
1






active

oldest

votes


















2














In the interests of moving this question off the unanswered queue, I'm converting my comment to an answer:



The splitting field is actually $mathbb Q(sqrt{7}i)$. Also, the second source is clearly wrong, since $2$ is a root.






share|cite|improve this answer





















    Your Answer





    StackExchange.ifUsing("editor", function () {
    return StackExchange.using("mathjaxEditing", function () {
    StackExchange.MarkdownEditor.creationCallbacks.add(function (editor, postfix) {
    StackExchange.mathjaxEditing.prepareWmdForMathJax(editor, postfix, [["$", "$"], ["\\(","\\)"]]);
    });
    });
    }, "mathjax-editing");

    StackExchange.ready(function() {
    var channelOptions = {
    tags: "".split(" "),
    id: "69"
    };
    initTagRenderer("".split(" "), "".split(" "), channelOptions);

    StackExchange.using("externalEditor", function() {
    // Have to fire editor after snippets, if snippets enabled
    if (StackExchange.settings.snippets.snippetsEnabled) {
    StackExchange.using("snippets", function() {
    createEditor();
    });
    }
    else {
    createEditor();
    }
    });

    function createEditor() {
    StackExchange.prepareEditor({
    heartbeatType: 'answer',
    autoActivateHeartbeat: false,
    convertImagesToLinks: true,
    noModals: true,
    showLowRepImageUploadWarning: true,
    reputationToPostImages: 10,
    bindNavPrevention: true,
    postfix: "",
    imageUploader: {
    brandingHtml: "Powered by u003ca class="icon-imgur-white" href="https://imgur.com/"u003eu003c/au003e",
    contentPolicyHtml: "User contributions licensed under u003ca href="https://creativecommons.org/licenses/by-sa/3.0/"u003ecc by-sa 3.0 with attribution requiredu003c/au003e u003ca href="https://stackoverflow.com/legal/content-policy"u003e(content policy)u003c/au003e",
    allowUrls: true
    },
    noCode: true, onDemand: true,
    discardSelector: ".discard-answer"
    ,immediatelyShowMarkdownHelp:true
    });


    }
    });














    draft saved

    draft discarded


















    StackExchange.ready(
    function () {
    StackExchange.openid.initPostLogin('.new-post-login', 'https%3a%2f%2fmath.stackexchange.com%2fquestions%2f3060179%2fgalois-group-of-x3-x2-4%23new-answer', 'question_page');
    }
    );

    Post as a guest















    Required, but never shown

























    1 Answer
    1






    active

    oldest

    votes








    1 Answer
    1






    active

    oldest

    votes









    active

    oldest

    votes






    active

    oldest

    votes









    2














    In the interests of moving this question off the unanswered queue, I'm converting my comment to an answer:



    The splitting field is actually $mathbb Q(sqrt{7}i)$. Also, the second source is clearly wrong, since $2$ is a root.






    share|cite|improve this answer


























      2














      In the interests of moving this question off the unanswered queue, I'm converting my comment to an answer:



      The splitting field is actually $mathbb Q(sqrt{7}i)$. Also, the second source is clearly wrong, since $2$ is a root.






      share|cite|improve this answer
























        2












        2








        2






        In the interests of moving this question off the unanswered queue, I'm converting my comment to an answer:



        The splitting field is actually $mathbb Q(sqrt{7}i)$. Also, the second source is clearly wrong, since $2$ is a root.






        share|cite|improve this answer












        In the interests of moving this question off the unanswered queue, I'm converting my comment to an answer:



        The splitting field is actually $mathbb Q(sqrt{7}i)$. Also, the second source is clearly wrong, since $2$ is a root.







        share|cite|improve this answer












        share|cite|improve this answer



        share|cite|improve this answer










        answered 2 days ago









        Cheerful Parsnip

        20.9k23396




        20.9k23396






























            draft saved

            draft discarded




















































            Thanks for contributing an answer to Mathematics Stack Exchange!


            • Please be sure to answer the question. Provide details and share your research!

            But avoid



            • Asking for help, clarification, or responding to other answers.

            • Making statements based on opinion; back them up with references or personal experience.


            Use MathJax to format equations. MathJax reference.


            To learn more, see our tips on writing great answers.





            Some of your past answers have not been well-received, and you're in danger of being blocked from answering.


            Please pay close attention to the following guidance:


            • Please be sure to answer the question. Provide details and share your research!

            But avoid



            • Asking for help, clarification, or responding to other answers.

            • Making statements based on opinion; back them up with references or personal experience.


            To learn more, see our tips on writing great answers.




            draft saved


            draft discarded














            StackExchange.ready(
            function () {
            StackExchange.openid.initPostLogin('.new-post-login', 'https%3a%2f%2fmath.stackexchange.com%2fquestions%2f3060179%2fgalois-group-of-x3-x2-4%23new-answer', 'question_page');
            }
            );

            Post as a guest















            Required, but never shown





















































            Required, but never shown














            Required, but never shown












            Required, but never shown







            Required, but never shown

































            Required, but never shown














            Required, but never shown












            Required, but never shown







            Required, but never shown







            Popular posts from this blog

            An IMO inspired problem

            Management

            Has there ever been an instance of an active nuclear power plant within or near a war zone?